Laplacian of the Euclidean Norm












2












$begingroup$


Let $r:mathbb{R}^n rightarrow mathbb{R}$ be defined by $r(x)=|x|,$ where $large|x|=(x_1^2+dots+x_n^2)^{frac{1}{2}}.$ Compute $nabla^2r,$ the Laplacian of $r.$





I perform the following computations,
$$largepartial_{x_i}r(x)=frac{1}{2} cdot frac{2x_i}{(x_1^2+dots+x_n^2)^{frac{1}{2}}}=frac{x_i}{r(x)},$$
so that we get
$$largepartial_{x_i}partial_{x_i}r(x)=frac{r(x)-frac{x_i^2}{r(x)}}{r(x)^2},$$
and putting these together yields
begin{align*}largenabla^2r(x)
&=largepartial_{x_1}^2r(x)+dots+partial_{x_n}^2r(x) \
&=largefrac{r(x)-x_1^2cdot r(x)^{-1}}{r(x)^2}+dots+frac{r(x)-x_n^2cdot r(x)^{-1}}{r(x)^2}\
&=largefrac{ncdot r(x)-r(x)^{-1}cdot r(x)^2}{r(x)^2}\
&=frac{n-1}{r(x)}.\
end{align*}



Is the above computation correct? I checked the calculation several times, but still have a feeling that I made a mistake somewhere










share|cite|improve this question











$endgroup$








  • 1




    $begingroup$
    Looks right to me
    $endgroup$
    – Anthony Ter
    Feb 3 at 9:50






  • 1




    $begingroup$
    Take a look to this document which addresses a more general question ; it confirms your result (see the last line !) staff.science.uu.nl/~kolk0101/Books/Analysis/normlinearmap.pdf
    $endgroup$
    – Jean Marie
    Feb 3 at 10:10
















2












$begingroup$


Let $r:mathbb{R}^n rightarrow mathbb{R}$ be defined by $r(x)=|x|,$ where $large|x|=(x_1^2+dots+x_n^2)^{frac{1}{2}}.$ Compute $nabla^2r,$ the Laplacian of $r.$





I perform the following computations,
$$largepartial_{x_i}r(x)=frac{1}{2} cdot frac{2x_i}{(x_1^2+dots+x_n^2)^{frac{1}{2}}}=frac{x_i}{r(x)},$$
so that we get
$$largepartial_{x_i}partial_{x_i}r(x)=frac{r(x)-frac{x_i^2}{r(x)}}{r(x)^2},$$
and putting these together yields
begin{align*}largenabla^2r(x)
&=largepartial_{x_1}^2r(x)+dots+partial_{x_n}^2r(x) \
&=largefrac{r(x)-x_1^2cdot r(x)^{-1}}{r(x)^2}+dots+frac{r(x)-x_n^2cdot r(x)^{-1}}{r(x)^2}\
&=largefrac{ncdot r(x)-r(x)^{-1}cdot r(x)^2}{r(x)^2}\
&=frac{n-1}{r(x)}.\
end{align*}



Is the above computation correct? I checked the calculation several times, but still have a feeling that I made a mistake somewhere










share|cite|improve this question











$endgroup$








  • 1




    $begingroup$
    Looks right to me
    $endgroup$
    – Anthony Ter
    Feb 3 at 9:50






  • 1




    $begingroup$
    Take a look to this document which addresses a more general question ; it confirms your result (see the last line !) staff.science.uu.nl/~kolk0101/Books/Analysis/normlinearmap.pdf
    $endgroup$
    – Jean Marie
    Feb 3 at 10:10














2












2








2





$begingroup$


Let $r:mathbb{R}^n rightarrow mathbb{R}$ be defined by $r(x)=|x|,$ where $large|x|=(x_1^2+dots+x_n^2)^{frac{1}{2}}.$ Compute $nabla^2r,$ the Laplacian of $r.$





I perform the following computations,
$$largepartial_{x_i}r(x)=frac{1}{2} cdot frac{2x_i}{(x_1^2+dots+x_n^2)^{frac{1}{2}}}=frac{x_i}{r(x)},$$
so that we get
$$largepartial_{x_i}partial_{x_i}r(x)=frac{r(x)-frac{x_i^2}{r(x)}}{r(x)^2},$$
and putting these together yields
begin{align*}largenabla^2r(x)
&=largepartial_{x_1}^2r(x)+dots+partial_{x_n}^2r(x) \
&=largefrac{r(x)-x_1^2cdot r(x)^{-1}}{r(x)^2}+dots+frac{r(x)-x_n^2cdot r(x)^{-1}}{r(x)^2}\
&=largefrac{ncdot r(x)-r(x)^{-1}cdot r(x)^2}{r(x)^2}\
&=frac{n-1}{r(x)}.\
end{align*}



Is the above computation correct? I checked the calculation several times, but still have a feeling that I made a mistake somewhere










share|cite|improve this question











$endgroup$




Let $r:mathbb{R}^n rightarrow mathbb{R}$ be defined by $r(x)=|x|,$ where $large|x|=(x_1^2+dots+x_n^2)^{frac{1}{2}}.$ Compute $nabla^2r,$ the Laplacian of $r.$





I perform the following computations,
$$largepartial_{x_i}r(x)=frac{1}{2} cdot frac{2x_i}{(x_1^2+dots+x_n^2)^{frac{1}{2}}}=frac{x_i}{r(x)},$$
so that we get
$$largepartial_{x_i}partial_{x_i}r(x)=frac{r(x)-frac{x_i^2}{r(x)}}{r(x)^2},$$
and putting these together yields
begin{align*}largenabla^2r(x)
&=largepartial_{x_1}^2r(x)+dots+partial_{x_n}^2r(x) \
&=largefrac{r(x)-x_1^2cdot r(x)^{-1}}{r(x)^2}+dots+frac{r(x)-x_n^2cdot r(x)^{-1}}{r(x)^2}\
&=largefrac{ncdot r(x)-r(x)^{-1}cdot r(x)^2}{r(x)^2}\
&=frac{n-1}{r(x)}.\
end{align*}



Is the above computation correct? I checked the calculation several times, but still have a feeling that I made a mistake somewhere







calculus multivariable-calculus partial-derivative






share|cite|improve this question















share|cite|improve this question













share|cite|improve this question




share|cite|improve this question








edited Feb 3 at 10:11









Jean Marie

31.5k42355




31.5k42355










asked Feb 3 at 9:33









Gaby AlfonsoGaby Alfonso

1,2051418




1,2051418








  • 1




    $begingroup$
    Looks right to me
    $endgroup$
    – Anthony Ter
    Feb 3 at 9:50






  • 1




    $begingroup$
    Take a look to this document which addresses a more general question ; it confirms your result (see the last line !) staff.science.uu.nl/~kolk0101/Books/Analysis/normlinearmap.pdf
    $endgroup$
    – Jean Marie
    Feb 3 at 10:10














  • 1




    $begingroup$
    Looks right to me
    $endgroup$
    – Anthony Ter
    Feb 3 at 9:50






  • 1




    $begingroup$
    Take a look to this document which addresses a more general question ; it confirms your result (see the last line !) staff.science.uu.nl/~kolk0101/Books/Analysis/normlinearmap.pdf
    $endgroup$
    – Jean Marie
    Feb 3 at 10:10








1




1




$begingroup$
Looks right to me
$endgroup$
– Anthony Ter
Feb 3 at 9:50




$begingroup$
Looks right to me
$endgroup$
– Anthony Ter
Feb 3 at 9:50




1




1




$begingroup$
Take a look to this document which addresses a more general question ; it confirms your result (see the last line !) staff.science.uu.nl/~kolk0101/Books/Analysis/normlinearmap.pdf
$endgroup$
– Jean Marie
Feb 3 at 10:10




$begingroup$
Take a look to this document which addresses a more general question ; it confirms your result (see the last line !) staff.science.uu.nl/~kolk0101/Books/Analysis/normlinearmap.pdf
$endgroup$
– Jean Marie
Feb 3 at 10:10










1 Answer
1






active

oldest

votes


















2












$begingroup$

If $u(x)=v(|x|)$ then
$$Delta u=frac{n-1}{|x|}v'(|x|)+v''(|x|).$$
See, for example https://web.stanford.edu/class/math220b/handouts/laplace.pdf






share|cite|improve this answer









$endgroup$














    Your Answer








    StackExchange.ready(function() {
    var channelOptions = {
    tags: "".split(" "),
    id: "69"
    };
    initTagRenderer("".split(" "), "".split(" "), channelOptions);

    StackExchange.using("externalEditor", function() {
    // Have to fire editor after snippets, if snippets enabled
    if (StackExchange.settings.snippets.snippetsEnabled) {
    StackExchange.using("snippets", function() {
    createEditor();
    });
    }
    else {
    createEditor();
    }
    });

    function createEditor() {
    StackExchange.prepareEditor({
    heartbeatType: 'answer',
    autoActivateHeartbeat: false,
    convertImagesToLinks: true,
    noModals: true,
    showLowRepImageUploadWarning: true,
    reputationToPostImages: 10,
    bindNavPrevention: true,
    postfix: "",
    imageUploader: {
    brandingHtml: "Powered by u003ca class="icon-imgur-white" href="https://imgur.com/"u003eu003c/au003e",
    contentPolicyHtml: "User contributions licensed under u003ca href="https://creativecommons.org/licenses/by-sa/3.0/"u003ecc by-sa 3.0 with attribution requiredu003c/au003e u003ca href="https://stackoverflow.com/legal/content-policy"u003e(content policy)u003c/au003e",
    allowUrls: true
    },
    noCode: true, onDemand: true,
    discardSelector: ".discard-answer"
    ,immediatelyShowMarkdownHelp:true
    });


    }
    });














    draft saved

    draft discarded


















    StackExchange.ready(
    function () {
    StackExchange.openid.initPostLogin('.new-post-login', 'https%3a%2f%2fmath.stackexchange.com%2fquestions%2f3098355%2flaplacian-of-the-euclidean-norm%23new-answer', 'question_page');
    }
    );

    Post as a guest















    Required, but never shown

























    1 Answer
    1






    active

    oldest

    votes








    1 Answer
    1






    active

    oldest

    votes









    active

    oldest

    votes






    active

    oldest

    votes









    2












    $begingroup$

    If $u(x)=v(|x|)$ then
    $$Delta u=frac{n-1}{|x|}v'(|x|)+v''(|x|).$$
    See, for example https://web.stanford.edu/class/math220b/handouts/laplace.pdf






    share|cite|improve this answer









    $endgroup$


















      2












      $begingroup$

      If $u(x)=v(|x|)$ then
      $$Delta u=frac{n-1}{|x|}v'(|x|)+v''(|x|).$$
      See, for example https://web.stanford.edu/class/math220b/handouts/laplace.pdf






      share|cite|improve this answer









      $endgroup$
















        2












        2








        2





        $begingroup$

        If $u(x)=v(|x|)$ then
        $$Delta u=frac{n-1}{|x|}v'(|x|)+v''(|x|).$$
        See, for example https://web.stanford.edu/class/math220b/handouts/laplace.pdf






        share|cite|improve this answer









        $endgroup$



        If $u(x)=v(|x|)$ then
        $$Delta u=frac{n-1}{|x|}v'(|x|)+v''(|x|).$$
        See, for example https://web.stanford.edu/class/math220b/handouts/laplace.pdf







        share|cite|improve this answer












        share|cite|improve this answer



        share|cite|improve this answer










        answered Feb 3 at 11:08









        Aleksas DomarkasAleksas Domarkas

        1,62317




        1,62317






























            draft saved

            draft discarded




















































            Thanks for contributing an answer to Mathematics Stack Exchange!


            • Please be sure to answer the question. Provide details and share your research!

            But avoid



            • Asking for help, clarification, or responding to other answers.

            • Making statements based on opinion; back them up with references or personal experience.


            Use MathJax to format equations. MathJax reference.


            To learn more, see our tips on writing great answers.




            draft saved


            draft discarded














            StackExchange.ready(
            function () {
            StackExchange.openid.initPostLogin('.new-post-login', 'https%3a%2f%2fmath.stackexchange.com%2fquestions%2f3098355%2flaplacian-of-the-euclidean-norm%23new-answer', 'question_page');
            }
            );

            Post as a guest















            Required, but never shown





















































            Required, but never shown














            Required, but never shown












            Required, but never shown







            Required, but never shown

































            Required, but never shown














            Required, but never shown












            Required, but never shown







            Required, but never shown







            Popular posts from this blog

            Can a sorcerer learn a 5th-level spell early by creating spell slots using the Font of Magic feature?

            Does disintegrating a polymorphed enemy still kill it after the 2018 errata?

            A Topological Invariant for $pi_3(U(n))$